atypicalquant
atypicalquant
  • 37
  • 317 931
Quant Interviews and how to ACE them!
Check out QuantGuide and use promo code ATYPICALQUANT for a free trial of premium access (for new users only)
Our sponsor, quantguide.io/, is the leading Quant Interview Prep Platform and offers hundreds of questions on probability, statistics, and more.
------------------
🔢 Support this channel: www.patreon.com/atypicalquant
🔢 Book recommendations:
atypicalquant.net/quant-books/
🔢 Buymeacoffee: www.buymeacoffee.com/atypicalquant
For business inquiries, please use the email address displayed in the about tab (in the channel overview).
------------------
Special thanks to our Patreons:
Shehryar Saroya
------------------
Book recommendations: atypicalquant.net/quant-books/
These should cover all the types of interview problems that one encounters at hedge funds such as Two Sigma (2sigma), Citadel, WorldQuant, BlackRock, Optiver, Jane Street, Akuna Capital, Hudson River Trading, etc.
------------------
These animations are largely made using manim: github.com/3b1b/manim. Special thanks to 3Blue1Brown and the manim community!
------------------
Social media:
Reddit: www.reddit.com/u/atypicalquant
#quant #trader #researcher #analyst #developer #quantitativeresearch #finance #interviewquestions #datascientist #faang #datascience #interviewpreparation #quantguide
#3blue1brown #manim
มุมมอง: 24 884

วีดีโอ

Fair Boarding | Quant Interview Questions
มุมมอง 18Kปีที่แล้ว
🔢 Support this channel: www.patreon.com/atypicalquant 🔢 Code and more: atypicalquant.net/ 🔢 Buymeacoffee: www.buymeacoffee.com/atypicalquant For business inquiries, please use the email address displayed in the about tab (in the channel overview). Question: What is an impartial way to deal with an overbooked flight? Special thanks to our Patreons: Shehryar Saroya Timecodes 00:00 - Question 00:4...
Circles in a Ramen Bowl | Quant Interview Questions
มุมมอง 11Kปีที่แล้ว
Check out QuantGuide and use promo code ATYPICALQUANT for a free trial of premium access (for new users only) Our sponsor, quantguide.io/, is the leading Quant Interview Prep Platform and offers hundreds of questions on probability, statistics, and more. 🔢 Support this channel: www.patreon.com/atypicalquant 🔢 Code and more: atypicalquant.net/quant-interview-questions/how-many-circles-will-you-c...
Non-Markovian Paper Tossing Challenge | Quant Interview Questions
มุมมอง 8Kปีที่แล้ว
🔢 Support this channel: www.patreon.com/atypicalquant 🔢 Code and more: atypicalquant.net/quant-interview-questions/non-markovian-paper-tossing-challange/ 🔢 Buymeacoffee: www.buymeacoffee.com/atypicalquant For business inquiries, please use the email address displayed in the about tab (in the channel overview). Question: You are tossing 300 pieces of crumpled paper into a bin. You miss the first...
Moments and Probabilities | Quant Interview Questions
มุมมอง 8Kปีที่แล้ว
🔢 Support this channel: www.patreon.com/atypicalquant 🔢 Code and more: atypicalquant.net/quant-interview-questions/probabilities-and-moments/ 🔢 Buymeacoffee: www.buymeacoffee.com/atypicalquant For business inquiries, please use the email address displayed in the about tab (in the channel overview). Question: X is a random variable, taking positive integer values and having: E[X] = 1, E[X^2] = 2...
Checking if a coin is Fair | Quant Interview Questions
มุมมอง 10Kปีที่แล้ว
🔢 Support this channel: www.patreon.com/atypicalquant 🔢 Code and more: atypicalquant.net/quant-interview-questions/unfair-coin-detection/ 🔢 Buymeacoffee: www.buymeacoffee.com/atypicalquant For business inquiries, please use the email address displayed in the about tab (in the channel overview). Question: You receive a coin that is either fair (with a 50% probability of getting heads) or unfair ...
Quick Pi Estimation | Quant Interview Questions
มุมมอง 6Kปีที่แล้ว
🔢 Support this channel: www.patreon.com/atypicalquant 🔢 Code and more: atypicalquant.net/quant-interview-questions/estimating-pi-quickly/ 🔢 Buymeacoffee: www.buymeacoffee.com/atypicalquant For business inquiries, please use the email address displayed in the about tab (in the channel overview). Question: Write a code that correctly estimates Pi up to 15 digits in under 1 second! Special thanks ...
Doubling the Data Set used in a Linear Regression | Quant Interview Questions
มุมมอง 7Kปีที่แล้ว
🔢 Support this channel: www.patreon.com/atypicalquant 🔢 Code and more: atypicalquant.net/quant-interview-questions/doubling-the-input-data-for-a-linear-regression/ 🔢 Buymeacoffee: www.buymeacoffee.com/atypicalquant For business inquiries, please use the email address displayed in the about tab (in the channel overview). Question: Before running a linear regression, our quantitative research int...
Unexpected Number Guessing Gamble | Quant Interview Questions
มุมมอง 6Kปีที่แล้ว
🔢 Support this channel: www.patreon.com/atypicalquant 🔢 Code & more: atypicalquant.net/quant-interview-questions/unexpected-number-guessing-gamble/ 🔢 Buymeacoffee: www.buymeacoffee.com/atypicalquant For business inquiries, please use the email address displayed in the about tab (in the channel overview). Question: Two numbers are randomly chosen from the closed interval [0,1]. Afterward, you se...
Random Summation until One | Quant Interview Questions
มุมมอง 5Kปีที่แล้ว
🔢 Support this channel: www.patreon.com/atypicalquant 🔢 Code and more: atypicalquant.net/quant-interview-questions/summation-until-one/ 🔢 Buy me a Coffee: www.buymeacoffee.com/atypicalquant For business inquiries, please use the email address displayed in the about tab (in the channel overview). Question: You play a game where you sum independent identically distributed uniform random variables...
Minimal Pairwise Correlation | Quant Interview Questions
มุมมอง 4.7Kปีที่แล้ว
🔢 Support this channel: www.patreon.com/atypicalquant 🔢 Code and more: atypicalquant.net/quant-interview-questions/minimal-pairwise-correlation/ 🔢 Buymeacoffee: www.buymeacoffee.com/atypicalquant For business inquiries, please use the email address displayed in the about tab (in the channel overview). Errata: At 3:45 (and onward) there's a typo on the first row, third column. Should be rho inst...
Expected Distances on a Unit Disc | Quant Interview Questions
มุมมอง 2.7K2 ปีที่แล้ว
🔢 Support this channel: www.patreon.com/atypicalquant 🔢 Code and more: atypicalquant.net/quant-interview-questions/expectation-of-distances-on-a-disc/ 🔢 Buymeacoffee: www.buymeacoffee.com/atypicalquant For business inquiries, please use the email address displayed in the about tab (in the channel overview). Question: What is the expected distance from a uniformly random point to its centre on a...
Random Variable Comparison | Quant Interview Questions
มุมมอง 4.7K2 ปีที่แล้ว
Random Variable Comparison | Quant Interview Questions
Card Draws needed for an Ace | Quant Interview Questions
มุมมอง 12K2 ปีที่แล้ว
Card Draws needed for an Ace | Quant Interview Questions
Logarithm Inequality | Quant Interview Questions
มุมมอง 6K2 ปีที่แล้ว
Logarithm Inequality | Quant Interview Questions
Build a South Park portfolio| Trading Basics
มุมมอง 3.4K2 ปีที่แล้ว
Build a South Park portfolio| Trading Basics
Airplane Boarding Problem | Quant Interview Questions
มุมมอง 10K2 ปีที่แล้ว
Airplane Boarding Problem | Quant Interview Questions
Fewer Coin Tosses, More Heads | Quant Interview Questions
มุมมอง 8K2 ปีที่แล้ว
Fewer Coin Tosses, More Heads | Quant Interview Questions
Number Guessing Game | Quant Interview Questions
มุมมอง 7K2 ปีที่แล้ว
Number Guessing Game | Quant Interview Questions
Cut a Cake with Planes: Maximum Number of Chunks | Quant Interview Questions
มุมมอง 5K2 ปีที่แล้ว
Cut a Cake with Planes: Maximum Number of Chunks | Quant Interview Questions
Squid Game: Survive the Bridge Game? | Quant Interview Questions
มุมมอง 8K2 ปีที่แล้ว
Squid Game: Survive the Bridge Game? | Quant Interview Questions
Efficient Algorithm for Finding the Mode | Quant Interview Questions
มุมมอง 5K2 ปีที่แล้ว
Efficient Algorithm for Finding the Mode | Quant Interview Questions
Tennis Tournament: Winner Probability | Quant Interview Questions
มุมมอง 7K2 ปีที่แล้ว
Tennis Tournament: Winner Probability | Quant Interview Questions
How much Leverage Should You Use? | Trading Basics
มุมมอง 3.1K2 ปีที่แล้ว
How much Leverage Should You Use? | Trading Basics
Treasure Division under Pirate Democracy: Game Theory | Quant Interview Questions
มุมมอง 7K2 ปีที่แล้ว
Treasure Division under Pirate Democracy: Game Theory | Quant Interview Questions
The EASIEST Math Olympiad Problem from 1985
มุมมอง 2K3 ปีที่แล้ว
The EASIEST Math Olympiad Problem from 1985
Getting Every Number on a Die | Quant Interview Questions
มุมมอง 19K3 ปีที่แล้ว
Getting Every Number on a Die | Quant Interview Questions
The EASIEST Math Olympiad Problem from 1984
มุมมอง 3.5K3 ปีที่แล้ว
The EASIEST Math Olympiad Problem from 1984
Coin Tosses needed for Consecutive Heads | Quant Interview Questions
มุมมอง 18K3 ปีที่แล้ว
Coin Tosses needed for Consecutive Heads | Quant Interview Questions
Sinaia - Cabana Piatra Arsa (via Transbucegi) in luna mai
มุมมอง 7K3 ปีที่แล้ว
Sinaia - Cabana Piatra Arsa (via Transbucegi) in luna mai

ความคิดเห็น

  • @JY-xz5to
    @JY-xz5to วันที่ผ่านมา

    Nice video. One minor thing: you choose alpha-level to be 0.1 to construct CI, but the resulting confidence level is 95%. I suppose it's because the criterion in making decisions is essentially one sided, so effective alpha-level is indeed 0.05. It might be a bit harder to sort this logic out.

  • @jc740
    @jc740 9 วันที่ผ่านมา

    Wow! I was always trying to understand this problem and you gave an amazing explanation!

    • @atypicalquant
      @atypicalquant 5 วันที่ผ่านมา

      Glad you liked it!

  • @amansinghal2431
    @amansinghal2431 9 วันที่ผ่านมา

    Genius!! The animation is simply flawless!!

    • @atypicalquant
      @atypicalquant 5 วันที่ผ่านมา

      Thank you! Happy to hear!

  • @amansinghal2431
    @amansinghal2431 9 วันที่ผ่านมา

    During 4:08 - 4:45, shouldn't it be Y21 instead of Y20 everywhere???

    • @atypicalquant
      @atypicalquant 5 วันที่ผ่านมา

      Hey! If you check how we defined Y_n at 2:30 and potentially also look into how P(X_1>=Y_1)=1/2 relates to the calculation at 1:59, you should see why we used Y20 at 4:08 - 4:45.

  • @john4556
    @john4556 หลายเดือนก่อน

    Can someone explain how at 4:00, the expression in the parentheses simplifies to (k+1)/2 ?

    • @atypicalquant
      @atypicalquant หลายเดือนก่อน

      The induction hypothesis is that each p_i =1/2. If you replace them with 0.5, you get the result.

  • @BanuRahuman
    @BanuRahuman 2 หลายเดือนก่อน

    isnt it 3Blue1Brown the TH-cam channel name?

    • @atypicalquant
      @atypicalquant หลายเดือนก่อน

      You're right, there is a typo in the video

  • @anujdubey6422
    @anujdubey6422 2 หลายเดือนก่อน

    IDK but the i am getting the right answer( atleast the integer part). Assume only 1 column. Hence, if wrong platform chosen, the answer will be N-1(people). If correctly chosen, the answer will be N. So the expectation is (N + N-1)/2 If there were 2 columns, the answer could have been (N + N-1 + N-2)/3 Generalize it, we will get 7.05 something. 🤔

  • @kesavanandm9109
    @kesavanandm9109 2 หลายเดือนก่อน

    An important and basic question that every quant must be able to answer. Thanks for posting. I think the random variable X is not the profit but the portfolio value at the end of k days. Return after k days must be (X - initial_value_of_portfolio)/initial_value_of_portfolio.

    • @atypicalquant
      @atypicalquant หลายเดือนก่อน

      You are correct, the return is the percentage change, and not the final value

  • @drunkabhi3697
    @drunkabhi3697 2 หลายเดือนก่อน

    I am unable to comprehend how the fairness of boarding is being considered in your solution as the person coming first has a probability of getting a seat = (k/k+n+1)^n vs for the last guy, its (k/k+n+1)

    • @atypicalquant
      @atypicalquant หลายเดือนก่อน

      I am unsure on why are you saying that the first person has a different probability of being chosen than the last. Which part of the induction proof is unclear?

    • @anirudhjanardhan788
      @anirudhjanardhan788 หลายเดือนก่อน

      i think the confusion comes from the fact that the first k people chosen to be seated in the airplane have to "survive" N-k rounds. In the example given with 4 people on the plane, it was implied that the 6th person who arrived would have the same chance to board the plane as the 5th, but as is pointed out in the algorithm, the random number is now generated from the set {1,2…6} so the 6th person has a 2/3 chance of boarding (if the value is 4 or lower they board). If you multiply out the probability of the first passenger "surviving" both rounds you’ll see it’s also 2/3, and this generalizes as more people attempt to board (4/5)*(5/6)*(6/7)…. So after N rounds each passenger has the same chance of boarding, namely k/N.

    • @atypicalquant
      @atypicalquant หลายเดือนก่อน

      Yes, @anirudhjanardhan788 that is the correct explanation!

    • @drunkabhi3697
      @drunkabhi3697 หลายเดือนก่อน

      Ah, yeah, I got it. Thanks for the explanations!

  • @hrishikeshmukherjee813
    @hrishikeshmukherjee813 2 หลายเดือนก่อน

    thanks

    • @atypicalquant
      @atypicalquant หลายเดือนก่อน

      You're welcome!

  • @ruturajpatwardhan9133
    @ruturajpatwardhan9133 2 หลายเดือนก่อน

    Superb solutions provided.

  • @ruturajpatwardhan9133
    @ruturajpatwardhan9133 3 หลายเดือนก่อน

    Thank you, I have supported through Thanks medium, hope you have received the payment. Keep going. 👍🏽👍🏽😀

  • @qiangzhang7367
    @qiangzhang7367 3 หลายเดือนก่อน

    Thanks. It is really clear. What if instead of duplicating the whole set, you just duplicate one data point?

    • @atypicalquant
      @atypicalquant หลายเดือนก่อน

      In the case of duplicating only one data point, the impact will be smaller, it will depend on the total number of data points and on the particular data point you choose to duplicate. It cannot be quantified as easily as duplicating the entire dataset.

  • @mathematics6199
    @mathematics6199 4 หลายเดือนก่อน

    How did you assume that Beta lies in multivariate normal distribution?

    • @atypicalquant
      @atypicalquant หลายเดือนก่อน

      This is just deduced from the assumptions of the OLS, mainly from the error term being normally distributed.

  • @mathematics6199
    @mathematics6199 4 หลายเดือนก่อน

    Actually, the rearringing part is somewhat tougher ig, rest is doable

    • @atypicalquant
      @atypicalquant หลายเดือนก่อน

      It might be, depend on how easily you can wrap your head around it.

  • @mathematics6199
    @mathematics6199 4 หลายเดือนก่อน

    Hello, I don't know few of the concepts listed here, like adjustment. I only know R squared, where can I check for?

    • @atypicalquant
      @atypicalquant หลายเดือนก่อน

      There are a few resources available to understand Rsquared adjusted, such as investopedia, or this wiki page: en.wikipedia.org/wiki/Coefficient_of_determination

  • @mathematics6199
    @mathematics6199 4 หลายเดือนก่อน

    Hi mam, I love your content!, could you please suggest where one can learn finance?

    • @atypicalquant
      @atypicalquant หลายเดือนก่อน

      I do have some resources for finance on my website, in the section of books I recommend

  • @adityakukreja7700
    @adityakukreja7700 5 หลายเดือนก่อน

    Thanks for this amazing video!

    • @atypicalquant
      @atypicalquant หลายเดือนก่อน

      Glad you liked it!

  • @CS_n00b
    @CS_n00b 5 หลายเดือนก่อน

    how do the beta coefficients vary when the data is duplicated for ridge regression?

    • @atypicalquant
      @atypicalquant หลายเดือนก่อน

      Hey! Great question. In the case of OLS, the coefficients stay unchanged as the changes we make end up simplifying nicely (see 4:00). However, for Ridge Regression, the closed form is w=(XX^T+λI)^(−1)Xy^T. Due to the extra lambda, coefficients will change (in a quite unpredictable way, from what I can tell). I've prepared a minimal working example here: colab.research.google.com/drive/1UKvPZwfCPGQWtumgyBcUGe3yyPeXNIvE?authuser=3#scrollTo=hoc9Xb9QuBvo&line=1&uniqifier=1

  • @nikolakv123
    @nikolakv123 5 หลายเดือนก่อน

    A solution I was able to do in 5-10 minutes is slightly different: Let E(x) be number of uniform(0,1) to go above x, for numbers smaller than 1. We quickly get a recursion E(x)=1+integral_from_0_to_x(E(t)dt). And guess e^x to avoid doing calculus 😅

  • @svetacodrean
    @svetacodrean 5 หลายเดือนก่อน

    For the formula of A(n) = (n+...+2+1)+A(1) = n(n+1)/2+1, shouldn't it be A(0) instead of A(1), since A(0)=1. Also, same goes for the formula S(n)=sum(i*(i+1)/2+1)+S(1). I believe it should be S(0). Correct me if I'm wrong.

    • @atypicalquant
      @atypicalquant หลายเดือนก่อน

      Hi! We arrived at the following recursive relation at 2:44, A(k+1) = A(k) + k+1 (*). For k=n-1, we get: A(n) = A(n-1) + (n-1)+1 = A(n-1) + n For k=n-2, we get: A(n-1) = A(n-2) + (n-2) + 1 = A(n-2) + n-1 ... For k=2, we get: A(3) = A(2) + 2 + 1 = A(2) + 3 For k=1, we get: A(2) = A(1) + 1 + 1 = A(1) + 2 ------------------------------------------------------------- (sum up and simplify) A(n) = n + n-1 + ... + 3 + 2 + A(1) Here, replacing A(1) with the 2 value from 1:25, you get that A(n) = n(n+1)/2+1. This was my approach. If I understand correctly, you propose that we also add this line For k=0, we get: A(1) = A(0) + 0 + 1 = A(0) + 1 And sum up and simplify after, leveraging A(0)=1. While I agree that this could be correct, it required some extra attention on defining A(0) as well as showing that (*) works for 0. I opted to avoid this :D

  • @mathematics6199
    @mathematics6199 5 หลายเดือนก่อน

    Hey, can you give the source of this question?

    • @atypicalquant
      @atypicalquant หลายเดือนก่อน

      This is question A4/2014 at the Putnam competition.

  • @laumatthew71
    @laumatthew71 5 หลายเดือนก่อน

    The proof of correctness for reservoir sampling here is incorrect. Consider we divide n elements into s groups and pick one group randomly. Each element is sampled with a probability s/n, but not every subset with size s will have an uniform probability to be chosen. The correct proof should relate with the Fisher-Yates shuffle

    • @atypicalquant
      @atypicalquant หลายเดือนก่อน

      Can you please add some more explanation on how dividing the n elements into s groups is related to the proof of reservoir sampling?

    • @laumatthew71
      @laumatthew71 หลายเดือนก่อน

      @@atypicalquant The algorithm (in this case, Reservoir Sampling) should achieve 2 goals: (i) Each element has a probability of s/n to be chosen (ii) ** Each subset of size s should have equal probability to be chosen I should not say the induction proof of wrong, but it is incomplete, as it only proofs point (i). why point (ii) is important ? Consider an alternative algorithm below: - We divide n elements into s groups (simply assume divisible) - Pick one group randomly - There are n/s groups, and we pick one randomly, with probability s/n - So, each element also has probability s/n to be chosen But if we perform the above "algorithm" repeatedly, if we pick an element "a" twice, all the other elements being picked together with "a" will be the same (which belong to the same group), which is not desired. So, we need something else to proof that Reservoir Sampling can indeed achieve goal (ii). Instead, we can relate the Reservoir Sampling with Fisher-Yates shuffle: If we consider the front s elements in a permutation is the "reservoir", then the two algorithms are equivalent. Since Fisher-Yates shuffle generates random permutation of n elements, the front s elements will be random subset of size s, having equal probability to be chosen, which proofed goal (ii) above.

    • @laumatthew71
      @laumatthew71 หลายเดือนก่อน

      @@atypicalquant The algorithm (in this case, Reservoir Sampling) should achieve 2 goals: (i) Each element has a probability of s/n to be chosen (ii) ** Each subset of size s should have equal probability to be chosen I should not say the induction proof of wrong, but it is incomplete, as it only proofs point (i). why point (ii) is important ? Consider an alternative algorithm below: - We divide n elements into s groups (simply assume divisible) - Pick only group randomly - There are n/s groups, and we pick one randomly, with probability s/n - So, each element also has probability s/n to be chosen But if we perform the above "algorithm" repeatedly, if we pick an element "a" twice, all the other elements being picked together with "a" will be the same (which belong to the same group), which is not desired. So, we need something else to proof that Reservoir Sampling can indeed achieve goal (ii). Instead, we can relate Reservoir Sampling with Fisher-Yates shuffle: If we consider the front s elements in a n-elements permutation to be the "reservoir", the two algorithms are equivalent. Since Fisher-Yates shuffle creates random permutations of n elements, for the first s spots is a random subset of size s, achieving our goal (ii)

    • @laumatthew71
      @laumatthew71 หลายเดือนก่อน

      @@atypicalquant The algorithm (in this case, Reservoir Sampling) should achieve 2 goals: (i) Each element has a probability of s/n to be chosen (ii) ** Each subset of size s should have equal probability to be chosen I should not say the induction proof of wrong, but it is incomplete, as it only proofs point (i). why point (ii) is important ? Consider an alternative algorithm below: - We divide n elements into s groups (simply assume divisible) - Pick only group randomly - There are n/s groups, and we pick one randomly, with probability s/n - So, each element also has probability s/n to be chosen But if we perform the above "algorithm" repeatedly, if we pick an element "a" twice, all the other elements being picked together with "a" will be the same (which belong to the same group), which is not desired. So, we need something else to proof that Reservoir Sampling can indeed achieve goal (ii). Instead, we can relate Reservoir Sampling with Fisher-Yates shuffle: If we consider the front s elements in a n-elements permutation to be the "reservoir", the two algorithms are equivalent. Since Fisher-Yates shuffle creates random permutations of n elements, for the first s spots is a random subset of size s, achieving our goal (ii)

    • @laumatthew71
      @laumatthew71 หลายเดือนก่อน

      @@atypicalquant The algorithm (in this case, Reservoir Sampling) should achieve 2 goals: (i) Each element has a probability of s/n to be chosen (ii) ** Each subset of size s should have equal probability to be chosen I should not say the induction proof of wrong, but it is incomplete, as it only proofs point (i). why point (ii) is important ? Consider an alternative algorithm below: - We divide n elements into s groups (simply assume divisible) - Pick only group randomly - There are n/s groups, and we pick one randomly, with probability s/n - So, each element also has probability s/n to be chosen But if we perform the above "algorithm" repeatedly, if we pick an element "a" twice, all the other elements being picked together with "a" will be the same (which belong to the same group), which is not desired. So, we need something else to proof that Reservoir Sampling can indeed achieve goal (ii). Instead, we can relate Reservoir Sampling with Fisher-Yates shuffle: If we consider the front s elements in a n-elements permutation to be the "reservoir", the two algorithms are equivalent. Since Fisher-Yates shuffle creates random permutations of n elements, for the first s spots is a random subset of size s, achieving our goal (ii)

  • @GordonHugenay
    @GordonHugenay 6 หลายเดือนก่อน

    I don't think this answer is true? Are you claiming that if you have four planes that fulfill your conditions, then they will always divide up the sphere into 15 chunks? I think I have such a configuration, and it only makes 14 chunks (and I actually didn't succeed in making 15). Does anybody have an explicit example of four planes making 15 chunks?

    • @atypicalquant
      @atypicalquant หลายเดือนก่อน

      Yes, choosing 4 planes according to the conditions will create 15 chunks. There is actually a very nice animation here: en.wikipedia.org/wiki/Cake_number

  • @GordonHugenay
    @GordonHugenay 6 หลายเดือนก่อน

    The time constraints in these videos are so frustrating for me. I was capable of solving this question, but I took like 5 to 10 minutes. Do you really only have 2 minutes in a typical quant interview?

    • @atypicalquant
      @atypicalquant หลายเดือนก่อน

      The time constraints are more of a guideline, used to compare the problems between them. In general, interviewers expect you to start moving towards a solution in this timeframe, but will not cut you off after 2 minutes. They might intervene and guide you towards a solution.

  • @bryantjp6522
    @bryantjp6522 6 หลายเดือนก่อน

    <3<3<3

  • @userpmzjah82
    @userpmzjah82 6 หลายเดือนก่อน

    why do you take inverse of the area and multipy it to the integral?

    • @atypicalquant
      @atypicalquant หลายเดือนก่อน

      The expected value of a variable is the integral of X * pdf(X). Since the pdf(X) has to integrate to 1, we need to scale it with the total area of the triangle

  • @berelaxed2181
    @berelaxed2181 6 หลายเดือนก่อน

    Mam where are you? We need mentors like you. Please come back and make videos. ❤❤

  • @4ngry4vian
    @4ngry4vian 6 หลายเดือนก่อน

    Quicker solution that doesn't require large sums and combinatorial identities: For each of the tiles, flip a coin to represent the guess of the first player to reach that tile. (It is ok that some coin flips are unusued if certain tiles are never reached, e.g. if everyone loses.) Let X represent the number of wrong guesses out of the 18 flips. The number of eliminated players is min(X, 16). (If there are more than 16 wrong guesses, then all 16 players were eliminated.) We know X ~ Binomial(18, 1/2) which has expectation 9, so we can compute the expected number of eliminated players by focusing on cases where X differs from min(X, 16). Speficially, when there were 17 wrong guesses, we overcounted by 1, and when there were 18 wrong guesses, we overcounted by 2. E[min(X, 16)] = E[X] - E[X - min(X, 16)] = E[X] - E[(X-16) * 1{X=17 or X=18}] = 18/2 - (1 * 18 / 2^18 + 2 * 1 / 2^18) = 9 - 20 / 2^18. So the expected number of survivors is 16 - E[min(X, 16)] = 7 + 20 / 2^18.

    • @atypicalquant
      @atypicalquant หลายเดือนก่อน

      Great solution!

  • @sahilchowdhury2229
    @sahilchowdhury2229 6 หลายเดือนก่อน

    Let log_2(3) = x. Then, as 2^(3/2) = sqrt(8) < 3, so log_2(3) > 1.5. We note the LHS is x + 1/x. The derivative of this is 1-1/x^2, which is positive on the interval (1, inf), so x + 1/x is increasing on (1, inf). Clearly, 1 < x. So, x + 1/x > 1.5 + 1/1.5 = 13/6.

    • @atypicalquant
      @atypicalquant หลายเดือนก่อน

      Great solution!

  • @raunakgupta4-yearb.tech.me278
    @raunakgupta4-yearb.tech.me278 6 หลายเดือนก่อน

    great work, keep it up

    • @atypicalquant
      @atypicalquant 2 หลายเดือนก่อน

      Thanks a lot!

  • @aceofjoy1099
    @aceofjoy1099 7 หลายเดือนก่อน

    I love it. thanks

    • @atypicalquant
      @atypicalquant 7 หลายเดือนก่อน

      Happy to hear! You're welcome 😀

  • @fragnet7673
    @fragnet7673 7 หลายเดือนก่อน

    Thank you! Great quant thingy

  • @vicentiubucingeni
    @vicentiubucingeni 7 หลายเดือนก่อน

    Cat de greu a fost urcarea? Crezi ca se poate si cu un 320 xdrive?

    • @atypicalquant
      @atypicalquant 7 หลายเดือนก่อน

      Salut. Desi ca motor si tractiune masina ar putea teoretic sa faca fata, eu personal nu as incerca. Daca mergi vara, dalele de pe sosea (se vad putin la 2:30) sunt foarte dure si inalte si iti vor scutura rau masina (mai ales daca ai niste jante mai mari) si poate chiar ti se va lovi scutul. Daca mergi iarna, nu cred ca vei putea urca pe drumul acoperit de zapada. Mai mult, din cate știu, nu ai Hill Descent Control pe seria 3. Chiar daca reusesti sa urci, nu va fi deloc usor coboratul foarte abrupt si poti ajunge într-o râpă.

  • @rayenbenhassen3450
    @rayenbenhassen3450 7 หลายเดือนก่อน

    Quant jobs are not as famous as those of software engineering or machine learning in terms of social media attention as they are not as accessible, which may be the reason why this channel never got the attention it deserves.

    • @atypicalquant
      @atypicalquant หลายเดือนก่อน

      This might be true about quant jobs. Still, I am happy about being able to help the small community we have built here.

  • @alish2950
    @alish2950 7 หลายเดือนก่อน

    You can actually see that log2(3) > 1.5 straight away. Like you said it's between 1 and 2 because 2<3<4. 3 is directly between 2 and 4 but because the log curve flattens off above 1, it will be closer to 2. Hard to explain what I mean but I can prove it formally.

    • @atypicalquant
      @atypicalquant 7 หลายเดือนก่อน

      Indeed one can use the concavity of the function log2(x) for this particular justification

  • @johnpaul-dh7lm
    @johnpaul-dh7lm 7 หลายเดือนก่อน

    i loved it

  • @srinivasaraokatadi1593
    @srinivasaraokatadi1593 8 หลายเดือนก่อน

    52/4 = 13?

    • @atypicalquant
      @atypicalquant 7 หลายเดือนก่อน

      That is not the correct answer. You can (re)watch the TH-cam short or find the full video on my channel page to learn why :D

  • @user-od8xl8ii8n
    @user-od8xl8ii8n 8 หลายเดือนก่อน

    To construct examples that achieve the lower bound, we can use Cholesky decomposition

  • @barelyrendered6254
    @barelyrendered6254 8 หลายเดือนก่อน

    I used some paper and graphed 2^x vs 3^x and eyeballed it for the right answer

    • @atypicalquant
      @atypicalquant 8 หลายเดือนก่อน

      This is such in an interesting approach! I'm sure this at least helps with the intuition of the answer, which you can then prove more rigorously.

  • @naco747
    @naco747 8 หลายเดือนก่อน

    Why don't you just say that the probability of hit on the 3rd toss is 1/2. And then multiply by the probability of hit in the 4th toss, which is 3/4 (since you nailed the previous one). And so on to complete 288 tosses (300 in total). You quickly see that the probability of nailing all 288 remaining tosses is just a division of factorials: 288!/299! Since factorial of 299 is 299!=299*288!, you cancel out 288! And end up with 1/299.

    • @RomekKromek
      @RomekKromek 8 หลายเดือนก่อน

      Do you have to nail all 298 remaining? Or the first 2 also count so you have 1 nail 1 miss and need to nail 297 out of 298 remaining?

    • @atypicalquant
      @atypicalquant 8 หลายเดือนก่อน

      For the total of 298 hits, you need to hit 297 out of the remaining 298, given that the first two are known to be a hit and a miss

  • @hyukhanh2715
    @hyukhanh2715 8 หลายเดือนก่อน

    Hello! Can you publish the Manim code you use to make this video? It would be a great help, thank you so much !!

    • @atypicalquant
      @atypicalquant 8 หลายเดือนก่อน

      Hey! Unfortunately, at this time, I am not open to sharing the source code. It wouldn't help much anyway, as it was hastily written and relying on quite a few custom classes. If you have specific manim issues, I am happy to try and assist you with them.

  • @patrickchen7162
    @patrickchen7162 9 หลายเดือนก่อน

    Can you help me with the understanding sorry I’m not very good math but I chanced upon your video. If considering the solution from the earlier parts, ABCDEDDDDE: mode would be D considering the calculations; A1 ?0 C1 ?0 E1 ?0 D1 D2 D3 D2 However, what if the letters are ABBACCA: am I wrong to calculate by; A1 ?0 B1 ?0 C1 C2 C1. The mode would be C which is wrong

    • @atypicalquant
      @atypicalquant 8 หลายเดือนก่อน

      You have correctly applied the algorithm to the example you have, and it indeed does not produce the correct result. This is because, as stated in the question, the algorithm works when the mode of the array has more than half of the frequency in the array. This is not the case for your example, giving that D has frequency 0.5.

  • @markomekjavic
    @markomekjavic 9 หลายเดือนก่อน

    For the general case - how do we know that f(n) = w * f(n-1) + 2? In the game reset branch we have the f(n) as well which here just disappears

    • @markomekjavic
      @markomekjavic 9 หลายเดือนก่อน

      my bad, I forgot to multiply with the probabilities of the branches

  • @jaliyahkane5127
    @jaliyahkane5127 9 หลายเดือนก่อน

    Why is it choose m and not n? Why did we select up positions first and not right?

    • @atypicalquant
      @atypicalquant 8 หลายเดือนก่อน

      You can try and solve the problem by choosing right positions first and you will find that it generates the same answer. Once we fix the up(or right positions), the remaining can be set in only one way.